LSAT and Law School Admissions Forum

Get expert LSAT preparation and law school admissions advice from PowerScore Test Preparation.

User avatar
 Dave Killoran
PowerScore Staff
  • PowerScore Staff
  • Posts: 5848
  • Joined: Mar 25, 2011
|
#79627
Complete Question Explanation
(The complete setup for this game can be found here: lsat/viewtopic.php?t=6157)

The correct answer choice is (A).

The condition in this question stem immediately affects J. From the fifth and seventh rules, J must connect to O and exactly one other island, namely L or M. If any island connected to M cannot also connect to O, and J is already connected to O, then J cannot connect to M and consequently J must connect to L. Accordingly, answer choice (A) is correct.
 Sourpatch
  • Posts: 4
  • Joined: Jul 03, 2020
|
#76748
I’m not sure how to get the right answer for this. Please help.
 Jeremy Press
PowerScore Staff
  • PowerScore Staff
  • Posts: 1000
  • Joined: Jun 12, 2017
|
#76810
Hi Sourpatch,

The best way to approach this question is to think about which island is most strongly affected by the local condition. That's island J.

We know that island J must be connected to O. So the local condition of the question ("no island that is directly connected by bridge with M is also directly connected by bridge with O") tells us right away that J cannot be connected with M. J also can never be connected with K, because K is only connected to one other island and that island must be M or O.

So now we know that J is connected to O, and is definitely NOT connected to M or K. But, from the initial rules, J must be connected to two islands. And the only extra island left as the second one J can be connected to is L. That gets us to answer choice A!

I hope this helps!

Jeremy
User avatar
 Bmas123
  • Posts: 22
  • Joined: Aug 24, 2022
|
#99055
Hi! Why could this not have been C? If K was connected to O, it would not have to be connected to M. We could stil have J connected to O and J could have connected to either J or L to complete the 2nd line J needed. L could connect to M to satisfy that condition. What am I missing as to why this is incorrect? I think the wording of the question itself is what is really confusing me, if someone could explain that, it would be very helpful. Thank you!
 Luke Haqq
PowerScore Staff
  • PowerScore Staff
  • Posts: 712
  • Joined: Apr 26, 2012
|
#99084
Hi Bmas123!

We know from the original rules that J is connected with exactly two islands. And we know from the last of the rules that O is one of these islands. That leaves us with one of K, L, or M as the second island to which J is connected. We can eliminate M given the local conditions of this question, leaving us with K or L.

We can eliminate K because of the fourth rule, "J, K, and L are each directly connected by bridge with one or both of M and O," and the sixth rule, "K is directly connected by bridge with exactly one island." K can't connect with J because K is only connected with one other island, and that other island is either O or M.

This leaves us with L as the only other island besides O that J can be connected to under the conditions in this question.

Get the most out of your LSAT Prep Plus subscription.

Analyze and track your performance with our Testing and Analytics Package.